Help 4 consecutive numbers divisible by 4

  • Thread starter Thread starter nat_tx
  • Start date Start date
  • Tags Tags
    Numbers
Click For Summary
The discussion centers on proving that the expression n(n^2-1)(n+2) is divisible by 4 for any integer n. Participants note that two of the four consecutive integers involved are always even, ensuring the product is a multiple of 4. The expression can be factored into (n-1)(n)(n+1)(n+2), highlighting that it represents the product of four consecutive integers. Whether n is even or odd, the product includes at least two even numbers, confirming divisibility by 4. The solution emphasizes the importance of factoring and recognizing the structure of the integers involved.
nat_tx
Messages
1
Reaction score
0
help! 4 consecutive numbers divisible by 4

Homework Statement




prove that for any integer n(n^2-1)(n+2) is divisible by 4??



Homework Equations





The Attempt at a Solution


i know two of them are even, but how do i actually prove this??

thanks
 
Physics news on Phys.org


If you know that two of them are even, then each is of the form 2a and 2b for some integers a and b. Then multiplying them together gives a multiple of 4.

You can start this by factoring the formula given. Expand it completely and then factor out n. Then factor the resulting third degree polynomial. One of the factors is (x+1). Your first factor was n, the second is n+1, so I think you see where this is going. Then after factoring completely you can find the solution.
 


If n is even, then so is (n+2), and the product is divisible by 4.

If n is odd, then (n^2-1)=(n-1)(n+1) is a product of even numbers, so again the whole thing is divisible by 4.
 


Try factoring n(n2-1)(n+2) and see if you notice something.
 


Bohrok said:
Try factoring n(n2-1)(n+2) and see if you notice something.
If you mean "notice that this is (n-1)(n)(n+1)(n+2), the product of 4 consecutive integers, I suspect, from the title of this thread, that he already knew that!
 
Question: A clock's minute hand has length 4 and its hour hand has length 3. What is the distance between the tips at the moment when it is increasing most rapidly?(Putnam Exam Question) Answer: Making assumption that both the hands moves at constant angular velocities, the answer is ## \sqrt{7} .## But don't you think this assumption is somewhat doubtful and wrong?

Similar threads

  • · Replies 3 ·
Replies
3
Views
950
Replies
7
Views
4K
  • · Replies 7 ·
Replies
7
Views
2K
  • · Replies 4 ·
Replies
4
Views
2K
  • · Replies 4 ·
Replies
4
Views
1K
Replies
31
Views
6K
  • · Replies 9 ·
Replies
9
Views
2K
Replies
14
Views
4K
  • · Replies 8 ·
Replies
8
Views
2K
Replies
3
Views
1K